Practice Exam Questions

Ace your homework & exams now with Quizwiz!

What is the FIRST thing social workers should do when attempting to evaluate institutionalized racism in an agency? A. Analyze the agency's policies and procedures B. Discuss the issue of fairness with the supervisor C. Facilitate research on the impact of agency policies D. Review policies that have been successful in other agencies

Answer = A Rationale: Analyzing policies and procedures (key A) is the FIRST action to take. Once the assessment has been completed, options (B), (C), and (D) may be appropriate actions. A social worker may facilitate research on the impact of agency policies (C) but not before completing an analysis (key A). It is important to understand agency practices FIRST.

A social worker at a family services agency has been working with a parent and an eight-year-old child. The child is enrolled in the same school as the social worker's child. On the first day of school, the social worker finds out that the children are assigned to the same classroom. What should the social worker do NEXT? A. Talk with the agency supervisor B. Request that one child be moved to another classroom C. Share information on confidentiality with the family D. Refer the family to another social worker

Answer = A Rationale: Before any action steps are taken, including options (B), (C), and (D) the social worker should meet with the agency supervisor (key A) to address the potential dual relationship and discuss alternatives.

A client informed a social worker that despite a court mandate the client does not intend to attend future sessions. What should the social worker do FIRST? A. Inform the client of the consequences of that decision B. Advise the judge of the client's decision C. Assist the client in identifying and clarifying goals D. Express disappointment in the client's decision

Answer = A Rationale: Clients have a right to make choices. Ensuring the client understands the consequences of the choice (key A) is what should happen FIRST. Advising the judge of the client's decision (B) and assisting in identifying and clarifying goals (C) may occur but would not be done FIRST. Expressing disappointment in the decision (D) seems judgmental on the part of the social worker.

A client begins services with a social worker. The client has multiple problems, including substance abuse and posttraumatic stress. The client recently became unemployed, and a close friend died two weeks earlier. What should the social worker do NEXT? A. Ask the client what is most troubling now B. Refer the client for substance abuse treatment C. Encourage the client to find employment D. Begin client-centered bereavement counseling

Answer = A Rationale: Consulting with the client (key A) is the only option that supports client self-determination and engagement. This should be done NEXT before taking additional steps. Making a referral for substance abuse treatment (B), encouraging the client to find a job (C), and beginning bereavement counseling (D) all draw conclusions before determining the client's perspective.

A social worker meets with a client to complete a biopsychosocial assessment. The social worker also speaks with the client's parents and probation officer to gather additional information. What term BEST describes the social worker's actions? A. Data triangulation B. Client debriefing C. Historical checking D. Audit trail

Answer = A Rationale: Data triangulation (key A) is used in assessment to compare and verify information from multiple sources. By speaking with the client's parents and probation officer, the social worker is using data triangulation. It is the only option that accurately describes the social worker's actions.

A social worker meets with a couple who describe frequent arguments at home. They are unable to answer specific questions about the conflicts and lack insight into their arguments. What is the MOST effective method of assessing the couple's communication skills? A. Observation of the couple discussing an issue of concern B. Videotaping of the couple in their home C. Review of client reflections in individual journals D. Review of client responses on a communication skills test

Answer = A Rationale: Directly observing the couple (key A) is the MOST effective way to assess the couple's communication skills and provides an opportunity for the social worker to immediately follow up with related questions. Options (C) and (D) do not assess the dynamics within the couple, which is the presenting problem. Videotaping (B) would be difficult to arrange and is not as effective as direct observation (key A).

A social worker provides mental health services to a client who is elderly and gradually driving less because of health problems. What concern is the client MOST likely to experience because of this change? A. Decreased social contacts B. Functional decline C. Impaired memory D. Medication noncompliance

Answer = A Rationale: Driving less will MOST likely result in fewer socialization opportunities (key A). Functional decline (B), impaired memory (C), and medication noncompliane (D) may occur but are not likely a direct result of driving less.

A social worker meets with a family who recently immigrated. The family believes that alcohol has health benefits. The parents purchase alcohol for their young children and permit them to become intoxicated. The parents tell the social worker that this is a custom in their country of origin. What should the social worker do FIRST? A. File a child abuse report with the appropriate authorities B. Encourage the family to practice their cultural norms C. Consult with the agency supervisor D. Research the customs of the family's country

Answer = A Rationale: Intoxication of young children is a behavior that requires the social worker to report the abuse to the appropriate authorities (key A). While (C) and (D) may be action taken by the social worker, they should not be done FIRST. Option (B) does not focus on the safety of the children.

A second grade student has repeated absences from school, an unclean appearance, and occasional bruises. The student has historically maintained average grades and does not exhibit behavior difficulties in the classroom setting. What should the school social worker do FIRST? A. Interview the student B. Contact a child protective services agency C. Develop an intervention plan D. Facilitate a behavioral observation

Answer = A Rationale: It is important for the social worker to directly interview the student (key A) FIRST before facilitating a behavioral observation (D) or intervening in any way. In terms of client-centered work, it is appropriate to interview a student of this age. Option (B) is not appropriate before further determining more details. For example, bruising may be normative for a student of this age. It may be important to develop an intervention plan (C) but not before FIRST talking with the student.

A parent brings a 13-year-old daughter to meet with a social worker because of concerns about recent changes in the daughter's behavior. The parent indicates that the daughter was playful, outgoing, and social until the last year, when she began to isolate herself at times. The parent is concerned about the daughter's choice of friends and feels that her personality seems to have completely changed. The social worker would FIRST want to determine whether the daughter is: A. Struggling with new developmental crises B. Being unconsciously influenced by early childhood issues C. Becoming addicted to drugs or alcohol D. Suffering from clinical depression

Answer = A Rationale: It is important to FIRST determine if the daughter's behavior is related to age appropriate healthy development (key A) before considering other issues such as the impact of early childhood issues (B), substance addiction (C), or clinical depression (D). While these may be factors, they should not be the social worker's initial focus.

What activity is least effective for the social worker when engaging clients in the beginning phase of treatment? A. Considering options for problem resolution with the client B. Exploring the client's perceptions of the problem C. Reducing the client's anxiety by providing support D. Clarifying the services that can be offered to the client

Answer = A Rationale: Options (B), (C), and (D) are all aspects of the first steps of engagement and discussion of the services the social worker provides. Considering options for problem resolution with the client (key A) occurs later in the problem-solving process, making it the least effective activity in engagement.

A social worker is assisting a client with problem resolution. The problem has been identified, solutions have been explored, and pros and cons have been considered. What is the NEXT step toward resolving the problem? A. Developing specific goals B. Establishing a time line for resolution C. Identifying termination criteria D. Highlighting the client's strengths

Answer = A Rationale: Since the problem has been identified and solutions discussed, developing goals (key A) is the NEXT step. Establishing a time line (B) and identifying termination criteria (C) occur after goals have been set. Highlighting the client's strengths (D) would have already occurred in the assessment phase.

A hospital social worker meets with the parents of a three-year-old child who has a seizure disorder. Because of their religious beliefs, the parents refuse a test to measure the child's brain activity. The medical team feels strongly that the child could be at imminent risk without the test results. What statement should guide the social worker's response? A. The child's right to medical treatment supersedes the parent's right to refuse treatment B. The parents' right to religious expression supersedes their child's medical needs C. The parents must first consent to treatment per hospital protocol D. The family's right to well-being may override laws and policies

Answer = A Rationale: Social work ethical principles support the idea that the potential imminent harm to the child is the primary factor in this situation. Therefore, the child's welfare overrides the parents' religious beliefs (key A).

A social worker receives notice that a close family member living out of state has become gravely ill. The social worker makes plans to go to stay with the relative for an indeterminate period of time. What should the social worker do FIRST? A. Arrange for ongoing coverage for clients B. Direct the office assistant to cancel all appointments C. Send a letter to clients explaining the situation D. Provide clients with a list of alternative resources

Answer = A Rationale: The professional code of ethics indicates that social workers have an ethical responsibility to arrange for service coverage when they are unavailable (key A). Cancelling all appointments (B) does not take care of clients needs. Sending letters explaining the situation (C) or providing a list of resources (D) may occur later but would not be done FIRST.

A social worker believes the actions of administrators at the school are detrimental to students. What should the social worker do FIRST? A. Discuss the concerns with school officials B. Prepare a report for the school board C. Educate the parents on advocacy skills D. Empower the students to self advocate

Answer = A Rationale: The social work code of ethics suggests that discussing concerns with school officials (key A) is the FIRST action to take. Preparing a report for the school board (B), educating parents about advocacy skills (C), and empowering student self-advocacy (D) are all possible actions to take, but only after (key A) has occurred.

When beginning family therapy, what should the social worker do FIRST? A. Establish rapport B. Identify needs and wants C. Highlight family problems D. Communicate hope

Answer = A Rationale: The social worker's FIRST task with any client is to establish rapport (key A). This needs to occur before any further action is taken.

In a couples therapy session, the husband says that he expects his wife to take care of the household chores because that is what his mother did. The wife exhibits frustration. The social worker assists the couple in understanding their function as a unit. What framework is the social worker MOST likely using? A. Structural family B. Self-psychology C. Solution-focused D. Psychoanalytic

Answer = A Rationale: The structural family framework (key A) is the only option that incorporates roles and functions of family members. Helping the couple manage expectations of roles in the relationship is MOST likely approached through a structural family lens. The solution-focused framework (C) emphasizes problem solving through analyzing times when behavior is problematic. Self-psychology (B) and psychoanalysis (D) are typically individually focused and would not MOST likely aid in understanding this couple's functioning as a unit.

A school-age boy is being beaten almost daily by his father, who has a drug and alcohol problem for which he receives no treatment. The boy solves his own school conflicts by punching and kicking younger children. What is the BEST description of this behavior? A. Identification with the abuser B. Hostility toward peers C. A conduct disorder D. A personality disorder

Answer = A Rationale: This item is a basic recall of cycle of abuse concepts and dynamics. Identification with the abuser (key A) is the BEST descriptor of the behaviors in this vignette. The boy may be hostile towards peers (B), but it is likely because of his identification with the abuser. A conduct disorder (C) is not indicated. A personality disorder (D) is not reasonable given the information provided.

A social worker agency director hosts a luncheon to meet with leaders of other community social services agencies. The goal of the director is to increase knowledge about community resources available to clients. What does this method exemplify? A. Networking B. Planning C. Coordination D. Consultation

Answer = A Rationale: This item requires an understanding of basic terminology. Because the director's goal is to increase knowledge about available services and not to take further action, networking (key A) is the correct term. Planning (B), coordination (C), and consultation (D) all involve further action and are incorrect terms.

Parents bring their five-year-old child to a family center for treatment. During the intake interview, the parents report that their child is defiant because the child does not listen to commands to be quiet when the older sibling is studying. The social worker explains that the child is not deliberately defiant but is unable to understand things from the point of view of others. The social worker is explaining: A. Egocentrism B. Syllogistic reasoning C. Reversibility D. Conversation

Answer = A Rationale: This item requires knowledge that age appropriate behaviors for a five-year-old child include egocentrism (key A). Syllogistic reasoning (B) refers to the process of deduction and is outside the developmental range of a five-year-old child. Reversability (C) and conversation (D) are concepts unrelated to the behaviors described.

During a regular weekly session with a client in recovery from chemical dependency, a social worker notes that the client appears irritated when asked the standard questions about the progress of recovery during the past week. After several minutes the social worker says, "You have been telling me that you still want to work on recovery issues, but today you are not participating in the process. What has changed?" What technique is the social worker using to encourage feedback from the client? A. Confrontation B. Reflection C. Interpretation D. Clarification

Answer = A Rationale: This item requires recall of techniques and their uses. Confrontation (key A) is a technique that points out a discrepancy between the client's earlier statement and current behavior.

A social worker at a clinic for military veterans meets with a client. The client requests help with finding a job. Military records show that the client was diagnosed with posttraumatic stress disorder when discharged from the military ten years ago. The client denies this diagnosis and refuses to be evaluated by the clinic psychiatrist. What should the social worker do FIRST? A. Respect the client's right to refuse services B. Focus on the client's presenting problem of obtaining a job C. Refuse services until the client accepts the psychiatric evaluation D. Refer the client to the local employment center

Answer = A Rationale: This professional code of ethics indicates that the client has the rigth to refuse services. Respecting that right (key A) is what should happen FIRST. While focusing on employment (B) and referring to an employment center (D) may occur later, they would not happen FIRST. Refusing to provide services until the client complies (C) undermines the client's right to self-determination.

When a social worker discovers that a client has been lying, what is the MOST helpful course of action? A. Determining what is motivating the client's dishonesty B. Confronting the client immediately with evidence of the dishonesty C. Ignoring the behavior for the present and focusing on where the client is D. Interpreting for the client the underlying meaning of the lie

Answer = A Rationale: Understanding the motivation of the client (key A) will assist with treatment. Ignoring the behavior and focusing where the client is (C) avoids the behavior. Immediately confronting (B) and interpreting for the client what the lie means (D) are interventions that the social worker could take. However before moving to intervention, it is MOST helpful for the social worker to understand the motivation (key A).

A hospital social worker assists a patient who is scheduled for discharge. The patient and family feel the patient needs more time to regain strength before returning home alone. What should the social worker do NEXT? A. Educate the family on assisting with patient care B. Arrange a meeting to discuss aftercare options C. Inform the patient and family about hospital rules D. Ask hospital administration to extend the discharge date

Answer = B Rationale: Arranging a meeting with involved parties to discuss options following discharge (key B) is the NEXT action. It is the only option related to determining appropriate steps following discharge. Educating the family (A), explaining hospital rules (C), and asking for an extension of discharge date (D) may occur later, based on the outcome of the discharge planning meeting (key B) but would not occur NEXT.

A mother brings her four-year-old boy to a clinic following referral by his pediatrician. He is aggressive with his mother and peers and has poor language skills. What should the social worker do FIRST? A. Recommend firmer discipline B. Explore how the child is disciplined C. Begin a behavioral program D. Refer the child for speech therapy

Answer = B Rationale: Assessment should occur before any intervention. Exploring how the child is disciplined (key B) is the only option that is not an intervention. While initiating a behavioral program (C) or referring to speech therapy (D) may be indicated, they would not occur FIRST. A recommendation for firmer discipline (A) is premature.

A social worker provides case management to a child with multiple disabilities. The family uses several service sources, including school social work, psychiatric consultation, and a family therapist. What is the BEST strategy for evaluating the child's progress? A. Use of family self-evaluation tools B. Periodic case review conferences C. Repeated psychiatric measurement D. Parental and educational reports

Answer = B Rationale: Because multiple services and disciplines are involved with the child and family, case review conferences (key B) are the BEST way to evaluate the child's progress. Self-evaluation tools (A), repeated psychiatric measurement (C), and parental and educational reports (D) are all evaluation tools that could potentially be useful but are not the BEST approach.

A social worker is working with a 16-year-old client who was sexually abused by a former stepparent. The social worker believes it is important for the client to be able to talk about the abuse in order to deal with the complicated feelings the client has already expressed. The client tells the social worker that talking about the abuse is painful and asks not to proceed in that way. What is the social worker's BEST response? A. Reiterate the importance of dealing with feelings through talking about the abuse B. Change the course of treatment as the client requests C. Refer the client to a support group to hear from others who have experienced abuse D. Engage the client's family to provide more consistent support

Answer = B Rationale: Changing the course of treatment as requested (key B) is respectful of the client's self-determination and is likely to engage the client. Options (A), (C), and (D) may be appropriate actions but are not BEST, as they do not allow the client a voice in the intervention process.

A single mother of two young children is in therapy with a social worker at an outpatient mental health clinic. She describes herself as isolated and overwhelmed in her roles as a parent. The social worker is of the opinion that the client could benefit from a referral to a parent support group at a local child welfare agency. In exploring this as a possible option, what should the social worker do FIRST? A. Arrange an appointment for the client with the group leader B. Consult with the client about whether additional services are desired C. Send the client's treatment summary to the group leader D. Contact the agency to explore the appropriateness of a referral

Answer = B Rationale: Consulting with the client (key B) is the only option that supports client self-determination and engagement. Determining the client's interest should be done FIRST before taking additional steps such as arranging an appointment (A), sending the treatment summary (C), or exploring referral appropriateness (D).

A social worker in an agency providing services to clients with a chronic mental health diagnosis wants to ensure that policies to engage clients are in place. What statement is consistent with this goal? A. Clients will develop realistic personal goals B. Clients are encouraged to attend monthly progress reviews C. Stereotypes about disabilities are unacceptable D. Employees will ensure that clients are safe at all times

Answer = B Rationale: Encouraging client attendance at monthly reviews (key B) is the only option that reflects engagement. Option (A) focuses on measurable outcomes. Options (C) and (D) both focus on staff behaviors and are not ways of engaging clients.

A social worker meets with a new client who was recently reunited with the spouse, who is an injured soldier. The client is struggling with assisting the spouse to cope with the injury, but the spouse responds with irritation. What should the social worker do FIRST? A. Avoid the topic of the injury B. Talk with the client about the situation C. Ask to meet with the spouse to assist with reintegration D. Refer the client and spouse for marital counseling

Answer = B Rationale: Gaining a better understanding of the situation (key B) is the FIRST action the social worker should take with a new client who expressing stress. Asking to meet with the spouse (C) and making referral for marital counseling (D) may both be reasonable actions, but only after the situation has been discussed with the client. Avoiding the topic of injury (A) is dismissing the client's concerns and would harm rapport with the new client.

A social worker is working with a parent who has admitted to yelling at a child when feeling overwhelmed by the demands of parenting. The social worker teaches the parent coping skills, but the behavior toward the child does not improve. The social worker should FIRST consider: A. That the parent lacks adequate motivation for change B. Assessing other stressors affecting the parent C. Focusing the change effort on the child's behavior D. Using cognitive-behavioral strategies with the parent

Answer = B Rationale: It is important to FIRST assess why the parent's behavior toward the child has not improved (key B). Considering that the parent lacks adequate motivation (A) involves reaching a conclusion before further assessment occurs. Focusing on the child's behavior (C) does not focus on the parent's ability to cope. Before determining if cognitive-behavioral strategies are indicated (D), further assessment (key B) is required.

A 21-year-old patient is approaching discharge from the hospital following a serious suicide attempt. The social worker meets with him before discharge to assess his emotional state. He will not answer any specific questions. The patient tells the social worker that he just needs to get home. What should the social worker do NEXT? A. Provide the patient with suicide hotline and mental health resources B. Contact the appropriate staff to explore involuntary commitment C. Chart the conversation before approving the discharge plan D. Explain to the patient the seriousness of the suicide attempt

Answer = B Rationale: Keeping in mind the protection and safety of the patient, exploring involuntary commitment (key B) is the NEXT step. Given the described behaviors, this patient remains at high risk for suicide. Providing community resources (A) is appropriate if the patient is not at risk and is participating with the social worker. Documenting the conversation (C) does not focus on the immediate safety concerns. Explaining the seriousness of the attempt to the patient (D) should have occurred earlier in the patient's hospitalization.

A social worker develops educational sessions for parents of adolescents. The sessions focus on how parents can intervene to reduce sexual exploitation of their children on the internet. What developmental need is MOST important for the social worker to discuss with the parents? A. Industry B. Intimacy C. Separation D. Autonomy

Answer = B Rationale: One of the primary tasks of adolescence is to develop intimate relationships with others outside the family unit. Sexual predators prey on adolescents who are seeking intimacy (key B). While (A), (C), and (D) may be related to sexual exploitation, they are not MOST important.

A social worker meets with a 15-your-old client who is promiscuous and struggles with low-self esteem. The client describes symptoms of an eating disorder. What should the social worker assess for FIRST? A. Oppositional defiant disorder B. Sexual abuse C. Suicidal ideation D. Adjustment disorder

Answer = B Rationale: Promiscuity, low self-esteem, and eating disorders are most strongly associated with sexual abuse (key B). There is nothing in the stem that would indicate oppositional defiant disorder (A) or suicidal ideation (C). Adjustment disorder (D) is a possibility, but is should not take precedence over sexual abuse as the FIRST area of concern.

A hospice social worker makes a home visit to a client who is becoming increasingly angry at his wife, who is his caregiver. The client is observed yelling and throwing books. What should the social worker do FIRST during the home visit? A. Refer the wife to a caregiver support group B. Assess the potential for further violence C. Instruct the wife to remove lethal means D. Evaluate home safety for hospice team members

Answer = B Rationale: Since the client has exhibited violent behaviors, it is important to assess his potential for further violence FIRST (key B). This will help ensure family safety. Referral to a caregiver support group (A) and instructing the removal of lethal means (C) may be appropriate but only after further assessment. Evaluating the home for safety for team members (D) should occur at some point but does not address the needs of the family unit.

A social worker is treating a young single mother for depression. The client expresses feeling overwhelmed while caring for her toddler and going to school. The client then reveals that she is pregnant and is planning an abortion. The social worker possesses strong negative feelings about the client's decision and feels unable to continue to treat her. What should the social worker do FIRST? A. Discuss the negative feelings with the client B. Obtain supervision about personal feelings C. Refer the client to another social worker D. Enter therapy to resolve the negative feelings

Answer = B Rationale: The ethical responsibility to obtain supervision (key B) is indicated because the social worker is having a strong negative reaction to the client's decision. Discussing the negative feelings with the client (A) is unethical and will likely harm the working relationship. Referring the client elsewhere (C) and entering therapy to resolve the feelings (D) may occur but only after obtaining supervision FIRST.

A social worker is interested in the social cohesion of a particular community. What should the social worker assess for FIRST? A. The rate of unemployment B. The level of trust in neighborhoods C. The high school graduation rate D. The social mobility within neighborhoods

Answer = B Rationale: The level of trust within the community (key B) is most closely linked to social cohesion and thus should be assessed FIRST. While unemployment rate (A), high school graduation (C), and social mobility within neighborhoods (D) are related to social cohesion, they would not be assessed FIRST.

A social worker meets with a client for a termination session. The client has frequently expressed gratitutde for the social worker's assistance during past sessions. During this session, the client presents the social worker with an expensive gift basket. After expressing gratitude, what should the social worker do NEXT? A. Accept the gift basket as a token of the client's appreciation B. Explain why the gift basket cannot be accepted C. Request that the gift be donated to another agency D. Make a gift of the basket to a client in need of the items

Answer = B Rationale: The social worker code of ethics indicates that social workers should avoid accepting gifts and services from clients (key B). Offering an explanation of why it cannot be accepted is respectful of the client's expression of gratitude. Option (A) is unethical. Asking the client to donate the gift to another agency (C) may be appropriate but not what would be done NEXT. Gifting the basket to a needy client (D) implies that the gift has been accepted, which is unethical.

A family applies to a local department of social services for assistance. The social worker uses established criteria to decide whether the family may receive financial assistance. What term MOST accurately describes what the social worker is doing? A. Family advocacy B. Eligibility determination C. Relief assistance D. Family planning

Answer = B Rationale: This item features a social worker determining a family's eligibility for services (key B). The social worker is not actively advocating for the family at this stage (A). Relief assistance (C) is not the MOST accurate term because the family has not been found to be eligible. Family planning (D) is not related to financial assistance.

During a group session, an adolescent has difficulty talking about an interaction with an abusive parent. The social worker helps the adolescent define and express feelings. What social work concept is demonstrated by this process? A. Summarizing B. Clarifying C. Reflecting D. Linking

Answer = B Rationale: This item requires an understanding of basic social work concepts. Defining and expressing feelings are key aspects of the skill of clarifying (key B).

A social worker is asked to devise a strategy for promoting greater community involvement. The social worker begins by facilitating a series of community meetings and focus groups to foster a shared sense of common needs and concerns. This approach to community intervention is MOST closely identified with: A. Policy advocacy B. Social planning C. Political activism D. Social movement

Answer = B Rationale: This item requires an understanding of macro-level concepts. Social planning (key B) is MOST closely related to the development of a shared sense of community. Options (A), (C), and (D) all include focused efforts on a common issue, which is not where the community is focused at this point.

A social worker is invited to a client's birthday celebration. In contemplating the invitation, what concept should the social worker consider? A. Enmeshed relationships B. Dual relationships C. Codependent relationships D. Triangulated relationships

Answer = B Rationale: This item requires knowledge of basic terminology. The concept of dual relationships (key B), as stated in the professional code of ethics, should be considered.

After the death of their child, a couple attend a bereavement group led by a social worker. The group meeting deals almost exclusively with this couple's loss. After the session, other group members report that this caused them to feel frustrated and uncomfortable. What should the social worker do NEXT? A. Ask the couple to join a different support group B. Plan to discuss the issue with the group at the next session C. Meet individually with the couple before the next session D. Remind the members of their own first group experiences

Answer = B Rationale: This item requires knowledge of group dynamics. Given that the group is the client, it is important to manage the concerns within the group by NEXT discussing the issue with the group (key B). Options (A), (C), and (D) may be appropriate steps to take depending on the discussion with the group (key B).

A 17-month-old child is placed in temporary foster care. Initially, the child cries vigorously, struggles in the arms of the foster mother, and will not be comforted. After a day or two, the child becomes increasingly quiet, withdrawn, and sad-looking. This is followed by an increase in activity, a decrease in crying, but little show of affect. The child's behavior can BEST be understood in terms of which of the following theories? A. Interpersonal B. Attachment C. Rational-emotive D. Ego development

Answer = B Rationale: This item requires knowledge of the components of attachment theory (key B). Interpersonal (A), rational-emotive (C), and ego development (D) theories do not address attachment issues such as those the child is experiencing.

What is the MOST practical research design for completing an agency program evaluation? A. Correlational B. Quasi-experimental C. Single-subject D. Qualitative

Answer = B Rationale: This item requires recall knowledge of research design.

A social worker is asked to email a client's records to a local physician. The client has signed a document legally permitting the release of the records. What should the social worker do NEXT? A. Email the records to the physician B. Hand deliver the records to the physician C. Confirm that the information can be securely transmitted D. Explain that ethical standards prohibit electronic transmission of information

Answer = C Rationale: As stated in the profession code of ethics, the social worker has the responsibility to ensure confidentiality of information transmitted through technology. Confirming that the information can be securely transmitted (key C) is the only option that respects the request for the records transfer and ensures that transfer will be confidential.

What is the PRIMARY barrier to women entering and completing substance abuse programs? A. Lack of access to programs B. Employment commitments C. Child care responsibilities D. Limited social support

Answer = C Rationale: Because women are the most common care providers of children, their child care responsibilities (key C) present the PRIMARY barrier to successful substance abuse program completion. While access (A), employment commitments (B), and limited social support (D) may be challenges, they are not the PRIMARY barrier.

A social worker provides services to a client at a mental health agency. The client dies suddenly. The client's family asks the social worker for the client's records. What should the social worker do NEXT? A. Consult with a colleague at the agency B. Refuse the family's request for records C. Determine the jurisdictional mandates about release of records D. Offer to meet with the family to help them with their loss

Answer = C Rationale: Before releasing any records, the social worker must determine jurisdictional mandates (key C). While consulting with a colleague (A) may occur, it would not be the NEXT action taken. Refusing the family's request for records (B) may occur but only after determining jurisdictional mandates (key C). Offering services to the family is not appropriate (D) and does not address the request for records.

A 16-year-old adolescent reveals to a social worker, in the course of being admitted to an inpatient psychiatric facility, that he uses alcohol on weekends and sniffs paint and gasoline on a daily basis. His family relationships are strained, his school performance is poor, and he has a driving violation. In the process of completing the assessment, what should the social worker recommend NEXT? A. Family sessions B. One-to-one observation C. Further psychological and neurological testing D. Review of school attendance and performance reports

Answer = C Rationale: Further psychological and neurological testing (key C) is warranted, given that the adolescent is being admitted to an inpatient treatment facility. Testing is immediately available and is needed to further assess the adolescent's condition and to rule out any underlying causes for the behaviors being exhibited. Family sessions (A) may be indicated after further testing. Observation (B) is important, but the seriousness and types of abuse that are reported indicate the need for further testing. Reviewing school reports (D) may provide supporting information but will not focus on the underlying cause of the adolescent's behaviors.

What is the MOST appropriate time for social workers in health care settings to gather family histories? A. During an acute medical crisis B. When directed by agency practice C. During the initial visit with the patient D. After meeting with a physician

Answer = C Rationale: History gathering should occur as early as possible in order for the social worker to gain a broad understanding of the elements impacting clients. The MOST appropriate time to gather family histories is during the initial visit with the patient (key C) as a part of the biopsychosocial assessment. Gathering family histories during an acute medical crisis (A) is not appropriate. It would be helpful for the social worker to have an understanding of family histories prior to meeting with the physician, so option (D) is incorrect. Agency practice (B) may be an important factor in the timing of history gathering but is not the MOST appropriate time to obtain this information from clients.

A social worker in a community agency servicing clients with HIV/AIDS is responsible for coordinating a variety of services to meet the clients' needs. After linking a client to programs to meet those needs, what should the social worker do NEXT? A. Develop a backup list of referral programs B. Reexamine the client's formal and informal resources C. Monitor the services received to ensure effectiveness D. Discharge the client from the community agency

Answer = C Rationale: In a care coordination model, the NEXT task after linking a client to needed program is to determine if services received have been effective (key C). Developing a list of referral programs (A), reexamining the client's resources (B), and discharging the client from the agency (D) may occur later in the process, but they would not happen NEXT.

In a routine review of case records at a mental health center, a social work supervisor discovers that case managers routinely refer all Native American/Indigenous peoples clients to either Alcoholics Anonymous or Al-Anon. In discussing this practice with the case managers, the supervisor learns that the referrals are made without obtaining information about the clients' use or abuse of substances. What should the supervisor do FIRST? A. Reprimand case managers about providing client services based on stereotyping B. Educate the staff about services provided by Alcoholics Anonymous and Al-Anon C. Provide staff training about culturally competent social work practice D. Allow no referrals to be made without approval from the supervisor.

Answer = C Rationale: It is important to provide educational interventions (key C) before moving to more restrictive supervisory practices. Options (A) and (D) are more restrictive and may be done later, after the provision of training has FIRST occurred. While staff members may benefit from being educated about Alcoholics Anonymous and Al-Anon (B), education about these services does not address the stereotyping that is occurring.

A client who is a college student expresses ambivalence about his sexual orientation during a session with a social worker. Despite having always dated women, the client now feels attracted to men. He is distressed and wants to better understand these feelings. What should the social worker do? A. Refer the client to a social worker who specializes in sexual orientation issues B. Explain that homosexuality is not a choice C. Examine the relationship between the client's concerns and his sexual identity D. Explore with the client how his family would react to his same-sex attraction

Answer = C Rationale: Since the client has expressed wanting to better understand his feelings, (key C) is the appropriate action for the social worker to take. Both (B) and (D) involve reaching conclusions without fully assessing the situation. Referring the client to another social worker (A) is inappropriate at this point.

A social worker is providing court mandated services to a child of a divorcing couple. The court has granted the father sole custody. The child's mother demands that the social worker share the child's records with her. What should the social worker do A. Schedule a session with both parents B. Disclose all records to the mother C. Provide the limitations of disclosure in writing D. Encourage the mother to talk with the father

Answer = C Rationale: The father is the child's legal guardian. Providing the limitations of disclosure in writing (key C) is the most responsible and ethical action for the social worker to take. The mother has no legal right to the child's records. Therefore, providing the records to her (B) is incorrect. Given the dynamics between the parents and the need for court mandated services, scheduling a session with both parents (A) and encouraging the mother to speak with the father (D) would likely not be productive.

A social worker refers a client to an agency for additional services that the social worker cannot provide. The social worker says, "Call me after your meeting to let me know what happened." What BEST describes the social worker's statement? A. The social worker is experiencing countertransference B. The social worker is assessing the quality of referral agencies C. The social worker is attempting to increase the client's chance of success D. The social worker is being overly protective of the client

Answer = C Rationale: The social worker is supporting action on the client's part by asking the client to call after the meeting. Increasing the client's chance of success (key C) will likely result. There is no indication that countertransference is occurring (A) or that the social worker is being overly protective (D). The social worker may be interested in assessing the quality of referral agencies (B), but it is not the BEST description of the statement.

What is the BEST definition of informed consent? A. An opportunity for the client to express expectations B. A client's decision at the beginning of the intervention process C. Decisions made by the client after receiving all relevant information D. A consensus process regarding information exchange and treatment

Answer = C Rationale: This is a recall item that requires knowledge of the definition of informed consent (key C).

During a session, a social worker becomes increasingly frustrated with a client's denial of issues. The social worker chooses to end the session early. What concept BEST fits this action? A. Boundary diffusion B. Disengagement C. Countertransference D. Role differential

Answer = C Rationale: This item requires an understanding of the definitions of basic concepts.

A social worker interviews an adolescent who recently emigrated from another country. The adolescent has abandoned cultural customs and refuses to speak his native language. What concept BEST describes this dynamic? A. Integration B. Separation C. Assimilation D. Marginalization

Answer = C Rationale: This item requires test-taker knowledge of the indicators of assimilation (key C).

Both the general public and decision makers have very little awareness of a community problem brought to the attention of a social worker. What strategy can BEST facilitate the development of policies or programs to correct the problem? A. Facilitating extensive client satisfaction surveys B. Organizing empowerment training for key players C. Using multiple forms of media communication D. Circulating a petition in the neighborhood

Answer = C Rationale: Using multiple media communication techniques (key C) BEST addresses the concern about community awareness. While facilitating satisfaction surveys (A) and empowerment training (B) may be steps later in the development of policies and programs, neither focuses on the specific concern. Circulating a petition (D) may educate the general public and decision makers but is not the BEST strategy.

A new social worker is working at a community mental health center with adult sex offenders. The social worker is assigned to a client who disgusts and repels him. When meeting with the client, the social worker finds himself unable to be sympathetic or empathetic. The social worker realizes that his intense feelings are not changing. After a month of discussion on this issue in supervision, there is still not improvement. What action should the social worker take NEXT? A. Discuss the feelings with a colleague without providing identifying details B. Explore whether this area of practice is appropriate for the social worker C. Pursue this issue in his own therapeutic treatment D. Ask to have the client transferred to another therapist

Answer = D Rationale: After a month of supervision on this issue, the social worker may be harming the client by continuing to provide services. Asking that the client be transferred to another social worker (key D) should occur NEXT. Options (A), (B), and (C) do not address the immediate need for the client to receive services.

A family member of a 54-year-old client tells the social worker that she is concerned about the client's behavior. The family member describes the client as having greater difficulty in managing activities of daily living and demonstrating a decline in memory and thought processing. What condition should the social worker assess the client for FIRST? A. Schizophrenia B. Alcohol use disorder C. Anxiety disorder D. Mild neurocognitive disorder

Answer = D Rationale: Difficulty in managing activities of daily living, memory decline, and impaired thought processing are most closely linked with mild neurocognitive disorder (key D). The combined symptoms are not consistent with options (A), (B), and (C).

A social worker in a family counseling agency has been asked to use a single-subject design with several clients to monitor their progress on treatment goals. The social worker is frustrated about the time required to update behavioral tallies and assessment instruments each week. Providing feedback to clients has also been much slower. What is the social worker's BEST approach to resolve this problem? A. Ask the supervisor to assign fewer clients B. Change to a program evaluation model of assessment C. Refuse to use session time for behavioral tallies D. Explore having clients document self-reported behaviors

Answer = D Rationale: Exploring the possibility of clients documenting self-reported behaviors (key D) focuses on engaging them in their treatment while also meeting the agency's expectations in using the research method. Options (A), (B) and (C) do not meet the agency's expecations for using single-subject research.

A social worker is a member of an advisory committee that is studying racially biased practices of the local police department. To raise community awareness of the issues, the committee has decided to sponsor a forum. Persons who have experienced problems will provide testimony. From an ethics perspective, what should the social worker do? A. Insist that committee members remain anonymous B. Invite the public to the forum C. Speak at the hearings about the committee's recommendations D. Ask speakers not to disclose confidential information in the hearings

Answer = D Rationale: Reminding speakers at the forum to maintain confidential information (key D) is an ethically based response. Social workers must support client rights to confidentiality in all settings, including public forums. Insisting that committee members remain anonymous (A) negates self-determination and choice. It also may not be helpful in building rapport with the community. Inviting the public to the forum (B) and speaking at hearings (C) do not take an ethics perspective into account.

A social worker who works in an assisted living facility meets with a resident prior to a family meeting. The resident says that his daughter is upset about his new romantic relationship. What is MOST important for the social worker to consider when planning the meeting? A. The daughter's resistance to the resident's choices B. The resident's attempts to maintain his independence C. The daughter's reaction to the romantic relationship D. The resident's feeling about his daughter's objections

Answer = D Rationale: The focus of the social worker is on the resident, who is the client in this situation. Understanding the resident's feelings about his daughter's objections (key D) is MOST important to consider. Options (A) and (C) focus on the daughter rather than the identified client. Option (B) indicates reaching a conclusion without understanding the dynamics of the relationship.

In a clinical setting, what is information gathered from an ecomap MOST often used for? A. To identify a family's evolutionary patterns B. To assess family strengths and weaknesses C. To assess intrafamilial communication patterns D. To identify systems that are part of a family's life

Answer = D Rationale: This is a basic recall of the uses of an ecomap (key D). Other tools are used to determine options (A), (B), and (C).

The wife and adolescent daughter of a man who is alcoholic find many reasons to deny and minimize his behavior and support his explanations of absences from work and frequent aggressiveness. What is the behavior of the wife and daughter called? A. Depersonalization B. Sublimation C. Avoidance D. Enabling

Answer = D Rationale: This is a recall question that requires knowledge of enabling behaviors (key D).

A client who experienced severe neglect by his biological parents during infancy is brought to a social work agency for evaluation. According to records, the client lived with several foster families before being adopted at the age of five. Despite receiving consistent nurturance from his adoptive family, the client has failed to bond with them. Upon initial assessment, a social worker notes that the client's inhibitions and ambivalence toward other could indicate the presence of: A. Rett syndrome B. Selective mutism C. Separation anxiety disorder D. Reactive attachment disorder

Answer = D Rationale: This item is a basic recall of the indicators of reactive attachment disorder (key D). Options (A), (B), and (C) could be reasonable, but the vignette points more directly to reactive attachment disorder (key D).

A hospital social worker investigates why a patient with home health services was admitted with a severe wound infection and malnutrition. The social worker calls the home health agency and talks with the person who ensures that the agency's obligations and ethical duties are being fulfilled and its rules are followed. What is this person's role in the agency? A. Paraprofessional B. Care planner C. Social activist D. Ombudsperson

Answer = D Rationale: This item requires a basic understanding of the roles of various professional positions in a home health care setting, including the ombudsperson (key D).

A social worker wants to assess the impact of heritage and belief systems on a family's interactions. What form of assessment BEST accomplishes this goal? A. Ecomap B. Genogram C. Person-in-environment D. Culturagram

Answer = D Rationale: This item requires basic knowledge of assessment tools and their uses.

What theory of addiction is used by Alcoholics Anonymous as the PRIMARY approach to explaining alcoholism? A. Psychodynamic B. Behavioral/social learning C. Biopsychosocial D. Disease

Answer = D Rationale: This question requires foundational knowledge of the theory underlying Alcoholics Anonymous, which views addiction as a disease (key D). While options (A), (B), and (C) may be used for interventions, the disease model (key D) is the only option used by Alcoholics Anonymous to explain addiction.

A social worker at a chemical dependency treatment center devotes times after each session to complete case notes. In recording the client's progress toward specific treatment goals, what information should the social worker include in each case note? A. Compliance with treatment program requirements B. The development of insight on the part of the client C. The client's level of investment in treatment D. The actual service or treatment provided during the session

Answer = D Rationale: While content of case notes varies from objective to subjective information, each case note should minimally include actual services provided during the session (key D). Options (A), (B), and (C) can be and are often included in the notes, but they do not always appear in each one.

What means of communication is MOST often misinterpreted by a non-Native American/Inigenous peoples social worker who is working with a traditional Native American/Inigenous peoples client? A. Body posture B. Eye contact C. Hand gestures D. Facial expressions

Answer = B Rationale: This item requires a test-taker to have basic knowledge about what literature suggests about traditional Native American/Inigenous peoples cultures and working in those communities. Eye contact (key B) is the correct answer.

A social worker is providing marital therapy to a couple. What goal statement is MOST likely to enhance a couple's motivation? A. Increase by one-fourth the frequency of positive statements B. Reduce by one-third the angry exchanges C. Eliminate all physical altercations D. Reduce by one-half the amount of shouting

Answer = A Rationale: The use of positive reinforcement is MOST likely to enhance motivation levels. Increasing the frequency of positive statements (key A) is the only option that is an example of positive reinforcement. Reducing or eliminating behaviors such as (B), (C), and (D) all involve reducing negative behaviors and reflect a deficit framework.

An agency social worker becomes aware that a coworker frequently consumes alcohol on their lunch break. When this occurs, the coworker does not appear intoxicated. This individual is responsible for driving foster children to and from specialized services throughout the day. What should the social worker do FIRST? A. Discuss the drinking directly with the coworker B. Inform the coworker's supervisor of the behavior C. Continue to observe and document the coworker's actions D. Consult the jurisdictional social work regulatory board for direction

Answer = A Rationale: As noted in the professional code of ethics, social workers should discuss the possible impariment directly with the colleague and assist with remedial action if appropriate (key A). Informing the supervisor (B) and consulting with the jurisdictional social work borad (D) may occur, but only after the social worker has addressed the concern with the colleague. Continuing to observe and document the coworker's actions (C) does not take into account the potential and immediate harm to clients that could result if the behavior continues.

A social worker is asked to evaluate a ten-year-old child who was reported to be talking with peers about explicit sexual activity and singing a song about incest. After ruling out direct sexual abuse to the child, what should the social worker do NEXT? A. Assess other exposures to sexually explicit material B. Assess the child's level of impulse control C. Consider a diagnosis of conduct disorder D. Educate the child about appropriate talk with peers

Answer = A Rationale: Assessing additional exposure (key A) is the NEXT reasonable action for the social worker to take. Direct sexual abuse may not be occurring, but the indicators noted suggest that the child is being exposed to some form of indirect sexual activity. The child's actions are not symptoms of conduct disorder (C) or poor impulse control (B). Educating the child (D) would be appropriate later but would not occur NEXT.

A former client contacts a social worker to request access to treatment records. The client and the client's spouse have been seen individually and jointly. What is the social worker's MOST appropriate response? A. Allow the client access to information pertaining only to the client B. Advise the client that agency policy prohibits client access to records C. Allow total access to records pertaining to the client and the spouse D. Request a ruling from the agency's executive director and attorney

Answer = A Rationale: Due to client confidentiality, only information pertaining directly to the client (key A) may be shared. Option (B) is unethical because records should be shared if reasonable. Allowing total access to all records (C) disregards the confidentiality of the spouse and is also unethical. Option (D) negates the social worker's responsibility to provide reasonable access to records and is not the MOST appropriate response.

A social worker meets with a new client who is diagnosed with depression. The social worker observes the client is ambivalent, non-trusting, and passive. What should the social worker do NEXT? A. Focus on directly engaging the client B. Suggest a task to improve the client's mood C. Encourage the client to recall better times D. Provide information about depression

Answer = A Rationale: Engaging the client (key A) is the only option that helps build rapport andsupports client self-determination. This should be done before taking additional steps such as suggesting a task to improve mood (B), encouraging recall of better times (C), and educating about depression (D).

A social worker meets with a client who is having marital difficulties. The client reports unmet needs and difficulty communicating the needs to the spouse. After assessing the nature of the relationship, what should the social worker do to BEST support skill development? A. Use role play as an intervention B. Encourage the client to be self-sufficient C. Refer the couple for marital counseling D. Provide reading materials on assertiveness

Answer = A Rationale: In the context of the session taking place, role play would be an appropriate intervention to assist the client in developing the skills to communicate more effectively with the spouse (key A). Options (B) and (C) deflect from the immediate concerns expressed by the client. Option (D) may support knowledge but not skill development.

A social worker leads a parenting skills group. During a session, a three-year-old child roars and gestures like a lion. The parent appears embarrassed and scolds the child for this behavior. What should the social worker do? A. Discuss the importance of pretend play B. Suggest the parent take the child to the zoo C. Demonstrate approval of the child's behavior D. Use an animal song to normalize the behavior

Answer = A Rationale: It is important to educate the parent about age appropriate healthy development. For a three-year-old child, this includes pretend play (key A). Suggesting a zoo visit (B) or using an animal song (D) does not address the parent's response to the child. Demonstrating approval of the child's behavior (C) also does not take into account the parent's reaction.

A social worker creates a job readiness program for young people transitioning to adulthood. The program is based on developmental theory. What is the PRIMARY goal of the program? A. Learning the basics of how to find employment B. Keeping track of the number of participants who secure employment C. Determining a particular career path at program onset D. Ensuring connections to community resources and systems

Answer = A Rationale: Learning how to find employment (key A) is the only option focusing on the developmental task of young adulthood and is the PRIMARY goal of the program. Tracking numbers of participants (B) is an evaluation of the program, not the goal. Determining a career path (C) may be an individual task but is not a program goal. Ensuring connections to community resources (D) may be a goal, but one that is secondary to learning how to find employment (key A).

A client who is trying to become pregnant uses marijuana consistently and has questions about the effects on the fetus. The social worker tells the client that the marijuana use could possibly harm the fetus. What should the social worker do NEXT? A. Offer substance abuse treatment options to the client B. Refer the case for possible criminal prosecution C. Inform the client that the drug use will be reported to the child protection agency D. Ask the client's health care provider to drug test the client at her appointments

Answer = A Rationale: Offering treatment options (key A) to the client supports client self-determination and choice. Options (B), (C), and (D) are all restrictive and breach client confidentiality.

A social worker facilitating change at a human services organization recognizes that clients are not assigned to the most apprpriate personnel. Case assignments are based only on caseloads, with no attempt to match clients' needs with workers' skill sets. On what aspect of the agency is the social worker focused? A. Organizational structure B. Agency mission C. Communication system D. Staff motivation

Answer = A Rationale: Organizational structure (key A) is the aspect of the agency that involves staff roles and responsibilities, including the process by which case assignments are based. Agency mission (B), communication system (C), and staff motivation (D), while related to agency services, do not focus on the identified issue of the process by which cases are assigned to staff.

What client factor MOST likely determines the scope and focus of an assessment? A. Needs of the client B. Motivation of the client C. Family systems D. Social network

Answer = A Rationale: The needs of the client (key A) must determine the focus of the assessment. Family systems (C) and social network (D) may be included in the assessment, but they should not be the primary factor for the assessment. Motivation (B) is more directly related to how the social worker would proceed in an intervention.

A social worker meets with a client who is chronically mentally ill. The client often refuses to take psychotropic medication but does not yet exhibit symptoms of active psychosis. What approach is MOST likely to help the client achieve long-term optimal functioning? A. Developing a behavioral program to enhance medication compliance B. Referring the client to a day treatment center C. Referring the client for long-term hospitalization D. Arranging for medication to be administered by a visiting nurse

Answer = A Rationale: The only approach MOST likely to help the client achieve long-term optimal functioning is medication compliance. Developing a behavioral program to assist the client in doing this (key A), rather than having the medication administered by a visiting nurse (D), better honors client self-determination and is less restrictive. Referral to a day treatment center (B) or for long-term hospitalization (C) are not MOST likely to help the client achieve long-term optimal functioning.

Termination of a 12-week therapy group needs to be initially discussed in the group: A. At the first session B. During the sixth session C. At the ninth or tenth session D. When the clients are ready

Answer = A Rationale: This item requires a basic understanding of group dynamics. Clarifying expectations and time frames are important issues to discuss at the beginning of a therapeutic group (key A). Both (B) and (C) are too late in the process to be effectivve. Discussing termination when clients are ready (D) does not acknowledge the group dynamic or clarify expectations.

What instrument is MOST likely to be used by social workers in the initial assessment of families? A. Ecomap B. Life history grid C. Family sculpting D. Global Assessment of Functioning Scale

Answer = A Rationale: This item requires knowledge of assessment tools and how they are used.

What is a pattern of alcohol and substance use that is more likely to be observed in women than in men? A. Abuse of prescription drugs B. Consumption of substances with friends C. Consumption of alcohol and drugs together D. Resistance to treatment referrals

Answer = A Rationale: This item requires test-taker knowledge of gender-based behaviors related to addiction.

A social worker is subpoenaed to testify at a court hearing. However, the client asks the social worker not to disclose information from their sessions. The social worker might support adhering to the client's request on the basis of: A. Privileged communication B. Self-determination C. Liability reduction D. Best practices

Answer = A Rationale: This item requires understanding of basic terminology used in social work practice.

A social worker in a high school is asked to facilitate a treatment group. What consideration would be MOST important in planning for the group? A. Conflict with authority is characteristic of adolescence B. Adolescents can form cliques C. The media can be influential on behavior at this age D. Drug and alcohol use should be addressed

Answer = A Rationale: This question requires knowledge of effective group facilitation and developmental tasks as defined by Erikson. While adolescents can form cliques (B), the media can be influential at this age (C), and drug and alcohol use (D) are all considerations, it is MOST important to recognize that there will be conflict with authority in an adolescent treatment group (key A).

A couple identify strongly with the deaf culture. A social worker is helping them to obtain education services for their child, who has experienced hearing loss. What communication strategy should the social worker consider during planning? A. Total immersion in the use of sign language B. Cochlear implant surgery to improve hearing C. Mainstreaming with children who do not have hearing loss D. The use of lipreading as a primary technique

Answer = A Rationale: This question requires knowledge of the deaf culture's belief in total immersion in their primary language, which is sign language (key A). Cochlear implant surgery (B) and lipreading (D) are strategies that are not supported by the deaf culture. Mainstreaming with children who do not have hearing loss (C) is not a communication strategy but an academic one.

A woman is referred to a domestic violence shelter after experiencing abuse by her partner. The woman explain to the social worker that she will return to her abusive spouse because it is God's will. What concept BEST explains her reasoning? A. Fatalism B. Nihilism C. Familism D. Paternalism

Answer = A Rationale: This question requires the test-taker to recognize demonstrations of fatalistic thought (key A): a belief that all events are predetermined and therefore inevitable.

The personality disorder in which the individual is hypersensitive to potential rejection, has low self-esteem, is socially withdrawn, and is generally unwilling to enter social relationships is: A. Avoidant B. Schizoid C. Antisocial D. Histrionic

Answer = A Rationale: This recall item requires basic knowledge of the features of personality disorders.

A social worker facilitates therapy with a married couple. Because of limited experience working with the couple's issues, the social worker approaches a colleague in the community for structured advice. What term BEST describes this action? A. Consultation B. Supervision C. Cultural competency D. Interdisciplinary collaboration

Answer = A Rationale: This requires a basic knowledge of the concept of professional consultation (key A).

A social work administrator in a public social service agency is assessing how to provide services in response to an influx of multiple immigrant groups. What should be the administrator's PRIMARY task? A. Ensuring coordinated services for all immigrant groups B. Establishing separate service centers for each ethnic group C. Hiring agency staff form immigrant groups D. Encouraging immigrants to assimilate into the larger society

Answer = A Rationale: To best provide services to multiple groups, the administrator should be PRIMARILY concerned with ensuring coordinated services for all (key A). Establishing separate service centers (B) may be too cost prohibitive and will not meet needs equitably across immigrant groups. While staffing from immigrant groups (C) may assist, the administrators PRIMARY responsibility is to establish what services are needed and in what framework. Encouraging assimilation into larger society (D) ignores cultural factors and will not assist in determining the best provision of services.

A medical social worker in a home health setting believes a client is abusing alcohol. The nurse reports that the client is not progressing medically. During a home visit, the social worker notices multiple empty liquor bottles in the trash. What should the social worker do FIRST? A. Explore this observation with the client B. Refer the client to chemical dependency treatment C. Assess the client for acute alcohol poisoning D. Request that the nurse make an additional visit

Answer = A Rationale: Before concluding that alcohol poisoning is occurring (C), intervening with a treatment referral (B) or making a referral to a team member (D), the social worker should discuss the observation with the client (key A). This is the FIRST step in engaging the client and learning more about the situation.

What practice has the MOST positive effect on service delivery? A. Using employment practices that support diverse staff skills B. Making the needs of clients the focus for all agency policy C. Providing paraprofessionals to ensure client attendance at all activities D. Providing professional development time for in-service training

Answer = B Rationale: Central to the mission of social work is the importance of clients being the center of service (key B). While options (A), (C), and (D) may support positive policies and practices, without a client-centered focus, they will not be effective.

A school social worker completes a functional behavioral assessment of a child who is exhibiting challenging classroom behaviors. What is the social worker trying to identify by completing this assessment? A. An overview of the client's family system B. An understanding of why the disruptions occur C. The strengths of the client D. Times when learning can best occur

Answer = B Rationale: Completing a functional behavioral assessment allows the social worker to understand the reasons for the child's disruptions (key B). It is the only option that is specifically used for this purpose.

During a therapy session, a client reveals to the social worker that she has difficulty managing her three-year-old child. The client reports that her child ran into traffic and was nearly hit by a car. What should the social worker do FIRST? A. Report the possible neglect to child protective services B. Work with the client to develop a safety plan C. Consult with the social work supervisor D. Refer the client to a parenting class

Answer = B Rationale: Developing a safety plan (key B) is the FIRST action that a social worker would take upon learning of the client's concerns. While the social worker might consult with a supervisor (C) and refer the client to a parenting class (D), these would occure only after attending to the client's immedicate issues. There is no evidence of child abuse or neglect (A).

A social work supervisor meets with a staff person who has difficulty setting limits with clients. What is the MOST effective strategy that the supervisor could suggest to correct this pattern? A. Avoid contact with manipulative clients B. Increase assertiveness through educational activities C. Use work time for self-appraisal D. Ask colleagues to provide feedback during client sessions

Answer = B Rationale: Increasing assertiveness through educational activities (key B) is the only option that directly focuses on improving the staff person's skills. Avoiding contact with clients (A) will not correct the pattern. While using work time for self-appraisal (C) and asking colleagues for feedback (D) could be used to support the staff person's development, the MOST helpful strategy is improving the staff person's skill level (key B).

A social worker supervises volunteers in a daycare facility with children from multiple ethnic backgrounds. Parents are complaining about one volunteer's lack of sensitivity. What should the social worker do FIRST? A. Assist the volunteer in working with the families B. Assess the volunteer's cultural competence C. Reassign the volunteer's responsibilities D. Recommend education workshops

Answer = B Rationale: It is important to FIRST assess (key B) before recommending interventions. Depending on the assessment findings, assisting in working with families (A), re-assigning responsibilities (C), and recommending further education (D) may be appropriate.

What is a social worker required to do before beginning work with a client that includes electronic transfer of records? A. Mail a copy of the requirements to all clients, including minors B. Provide the client with the agency privacy policy C. Give the client information regarding fee policies D. Obtain signed insurance releases to protect client privacy

Answer = B Rationale: It is the ethical responsibility of the social worker to ensure that clients understand the risks and limits of privacy when electronically transferring records (B). Maining a copy of the requirements to clients (A) is not required. Providing fee policy information (C) is unrelated to the electronic transfer of records. Obtaining insurance releases (D) would occur after ensuring that clients understood the agency's privacy policy (key B).

A social worker at an adoption agency receives a subpoena for the records of an adult client recently seen by the social worker regarding the client's adoption many years ago. In the process of responding to the subpoena, what should the social worker do FIRST? A. Submit the client's record as requested by the subpoena B. Discuss the subpoena with the agency's legal counsel C. Discuss the subpoena with a professional colleague D. Ask for approval from the client to respond to the subpoena

Answer = B Rationale: Once a subpoena is received, the social worker should FIRST discuss it with legal counsel (key B) before submitting it (A). The social worker may discuss the subpoena with a colleague (C) but not before obtaining legal counsel. Asking for approval from the client (D) before responding to the subpoena would occur only if legal counsel directed this.

A weekly meeting is held at a family services agency to determine if referred families meet the criteria for services. Staff members are frustrated because the meetings have become characterized by unrelated discussion and have begun to last several hours. What should the social worker do as chair of the meeting? A. Limit the number of cases presented B. Redirect staff members who digress C. Schedule individual meetings with problem staff members D. Set time limits for the meeting

Answer = B Rationale: One of the key roles of a group facilitator is to keep participants focused on the purpose of the meeting. Since staff members are discussing unrelated matters in the meeting, providing redirection is critical (key B). While limiting the number of cases presented (A), scheduling individual meetings (C), and setting metting time limits (D) may be appropriate interventions, they are not the most helpful options for the social worker to take in order to ensure that the meetings run smoothly and efficiently.

A social worker provides treatment to clients on the sex offender registry. Some clients are discouraged when they are fired from a job after their sex offender status becomes known. What should the social worker do to BEST support clients? A. Refer the clients to a human rights agency B. Teach clients how to disclose their background proactively when offered a job C. Require clients to empathize with how employers view individuals registered as sex offenders D. Discourage clients from pursuing employment until they complete treatment

Answer = B Rationale: Teaching clients how to proactively disclose their background (key B) offers the BEST support because it involves empowerment and demonstrates respect for the dignity and worth of the person. Referring clients to a human rights agency (A) would occur only after a potential human rights violation is identified. It is unrealistic to require empathy (C). Discouraging clients from pursuing employment until treatment is completed (D) does not promote self-determination and is not a supportive, strengths based approach.

A social worker is asked by community members to join a new task force. The goal of the task force is to prevent adolescent gang activity, which community members fear will happen soon. In what setting are adolescent gangs MOST likely to emerge? A. Near public schools B. In inner-city neighborhoods C. In economically depressed areas D. Near communities that do not enforce curfews

Answer = C Rationale: This item requires knowledge of the causal factors for gang formation. Gans are MOST likely to emerge in economically depressed areas (key C).

A social worker meets with an eight-year-old client to work on issues of impulsivity and anger management. The client tells the social worker of his intent to push his infant sibling down the stairs. The client asks the social worker not to tell anyone. After discussing the client's concerns, what should the social worker do NEXT? A. Report to the appropriate jurisdictional authorities B. Discuss the client's statements with the parents C. Assess the client's current coping strategies D. Teach anger management techniques

Answer = B Rationale: The social worker has a duty to warn the parents of potential harm to the infant sibling (key B). Assessment of current client coping strategies (C) and help with anger management techniques (D) are potential strategies for the social worker, but they are not something the social worker would do NEXT. Because no harm has occurred, reporting to authorities (A) would be inappropriate.

A social worker makes a home visit to a family to investigate alleged neglect of one of the children. The social worker spends the first part of the interview discussing the family's new roof, their garden, and the rising costs of school supplies. In doing this, the social worker is purposefully: A. Demostrating the importance of good conversational skills B. Employing an important warm-up period to establish rapport C. Disarming the family so that they will be straightforward in responding D. Avoiding discussion of a sensitive topic

Answer = B Rationale: The social worker is using the technique of small talk to begin establishing a relationsip with the family and to build rapport (key B). While this activity may also demostrate good conversational skills (A), it is not the purpose for the social worker's actions. Both disarming the family (C) and avoiding a sensitive topic (D) are contrary to the purpose of building rapport and will not serve to fully engage the family.

A social worker is employed by a primary care clinic that offers services to a local Native American/Indigenous peoples tribe. Some tribe members have reported reluctance to use the health care services even though they recognize the potential benefits. What is the social worker's BEST response? A. Provide education on Western medicine B. Include tribal healers in the clinic setting C. Use tribal folklore while providing clinic services D. Self-disclose personal family health history

Answer = B Rationale: The social worker must address the needs of the population being served and, in doing so, must acknowledge the ways in which cultural health beliefs can affect service delivery. There is no indication that the tribe members are uneducated about Western medicine (A), and disclosing personal health history (D) is irrelevant. Using tribal folklore while providing clinic services (C) could be incorporated, but including tribal healers in the clinic setting (key B) BEST demonstrates respect for cultural beliefs and the importance of trust in health care.

What task is the FIRST step in setting up a program evaluation? A. Selecting a criteria for measuring success B. Determining the goals and objectives C. Assessing the available resources D. Designing the methodology

Answer = B Rationale: This is a basic recall item of steps necessary in setting up a program evaluation. The goals and objectives (key B) must be completed FIRST. Later steps could include selecting ways to measure success (A), assessing available resources (C), and designing the methodology (D).

What personality disorder is characterized by a restricted range of emotional experience and expression? A. Antisocial B. Schizoid C. Histrionic D. Dependent

Answer = B Rationale: This is a basic recall question requiring knowledge of personality disorders.

A hospital social worker works with a terminally ill patient who is refusing the use of a breathing machine to prolong life. What action is the patient choosing? A. Active euthanasia B. Natural death C. Physician-assisted death D. Palliative care

Answer = B Rationale: This item requires a basic understanding of medical terms related to end of life care.

A social worker facilitates a group for domestic violence perpetrators. A group member is now able to recognize the triggers for anger and is committed to changing resulting reactions. According to the transtheoretical model of change, what stage is the group member MOST likely experiencing? A. Precontemplation B. Contemplation C. Maintenance D. Termination

Answer = B Rationale: This item requires knowledge of the transtheorectical model of change and its stages. Because the group member recognizes the triggers and is committed to change but has not determined action steps, contemplation (key B) is the MOST likely stage.

During what stage does a gay or lesbian adolescent face the greatest need of treatment and the MOST stress? A. Becoming involved in an intimate same-sex relationship B. Coming out and acknowledging gay feelings C. Exploring gay identity D. Experiencing the sense of being different

Answer = B Rationale: This item requires the test-taker to have basic knowledge of the stages of gay identity development.

A social worker is asked to analyze new social welfare policies that will affect the community. What should the social worker do FIRST? A. Examine the likely impact of the policies on the community B. Research the historical problems that led to the policies C. Organize a grassroots coalition to discuss reaction to the policies D. Look into legal precedents that could potentially nullify the policies

Answer = B Rationale: This item requires understanding of the basic steps in policy analysis. The social worker must FIRST understand the historical background of the policy (key B) before moving to the analysis stage. Options (A), (C), and (D) are steps to take later in the process.

A preschooler's parents report that their child masturbates at night and has tried to touch a sibling's genitals while bathing. What do these behaviors MOST likely indicate? A. Child sexual abuse B. Normal sexual develoment C. Diffuse personal boundaries D. Delayed superego development

Answer = B Rationale: This question requires an understanding of healthy sexual development (key B) for a preschooler.

A social worker working in a homeless shelter is reviewing policies and procedures. The social worker wants to focus on what can be done to promote client trust and mitigate power imbalances. What should the social worker do FIRST? A. Implement a staff training program on principles of confidentiality and exceptions B. Create a client-managed review board that investigates critical incidents C. Interview clients to learn ways of implementing policy to be sensitive to client needs D. Recruit former shelter clients to perform in policy advocacy roles

Answer = C Rationale: Since the focus is on developing client trust, the only option that accomplishes this is interviewing clients to learn ways of implementing policy sensitive to their needs (key C). Options (A), (B), and (D) are action steps that should be taken only after client input is obtained.

A school social worker meets with parents of a child who has recently completed treatment for cancer. The child's physician is recommending that the child return to school as soon as possible, but the parents believe that more time at home would be beneficial. What should the social worker do NEXT? A. Advocate for the parents to keep the child at home B. Explore the reasons for the parents' beliefs C. Obtain the child's perspective on the situation D. Refer the parents and child for family therapy

Answer = B Rationale: Before taking action, the social worker should acknowledge and validate the parents; concerns and explore the events and emotions that have led to these concerns (key B). Advocating that the child stay at home (A) is not supported by the physician's recommendation. Although the social worker may eventually want to obtain the child's perspective (C), this would not be the NEXT step. There are no indications of family dysfuction that would warrant family therapy (D).

A social worker who recently granduated begins working in a drug treatment program. The social worker notices that clients have many needs, including medical attention, mental health services, social services, housing assistance, and employment training. The social worker feels overwhelmed. What should the social worker do FIRST? A. Establish priorities B. Be aware of countertransference C. Obtain supervision D. Plan to write a grant to hire an additional worker

Answer = C Rationale: Because the social worker is feeling overwhelmed, obtaining supervision (key C) is necessary. Establishing priorities (A) and being aware of countertransferece (B) may occur, but not FIRST. Option (D) assumes that the problem can be resolved with more staffing. It may simply be the social worker's experience level that is causing the stress. Obtaining supervision (key C) FIRST will help the social worker to understand the situation.

A couple meet with a social worker about concerns in their relationship. Both partners report the other person does not offer emotional support or affection. What should the social worker explore FIRST? A. Gender roles B. Safety risks C. Relationship expectations D. Relationship development

Answer = C Rationale: Both partners are expressing a lack of emotional support. By FIRST exploring relationship expectations (key C), the social worker is focusing on the presenting problem. Exploring gender roles (A) and safety risks (B) may be important but makes assumptions about the relationship that are not known at this point. Relationship development (D) may occur later but is not the FIRST area of exploration.

A client meets with a social worker to discuss parenting a rebellious adolescent. After several sessions, the client shares religious beliefs that support physical punishment. What should the social worker do FIRST? A. Inform the client that physical punishment psychologically harms children B. Validate the client for openly sharing the physical punishment viewpoint C. Request that the client describe the physical punishment practices D. Explore the client's own history of physical punishment during childhood

Answer = C Rationale: Clarifying the client's basic understanding of physical punishment practices (key C) is a necessary FIRST step before any other action. Options (A), (B), and (D) are all actions that may occur later but not before understanding the client's perspective.

A client requests mental health services to manage chronic and escalating anger. During the initial session, the client tells the social worker of instances when she was verbally abusive to an individual in her care who uses a wheelchair. The client is remorseful and says that the incidents have occurred in only two to three occasions. What should the social worker do NEXT? A. Collect collateral information from the care recipient B. Explain the need to report the incidents to the proper authorities C. Discuss the incidents further to determine the appropriate action D. Develop a personalized anger management program for the client

Answer = C Rationale: During the initial session, the social worker should assess further (key C) before taking action such as collecting collateral information (A) or developing an anger management program (D). Explaining the need to report (B) may need to occur later, based on what is learned in the discussion (key C).

A child protective services social worker meets with a parent who has allegedly been neglectful. The parent talks about being overwhelmed by the needs of her child. She described having trouble sleeping because she needs to stay alert for messages from a favorite talk show host. The parent also shares beliefs that the child's teacher is against her. What should the social worker do NEXT? A. Refer the parent to a physician B. Encourage the parent to discuss her anxiety C. Initiate a mental status examination D. Plan a collaborative meeting with the school

Answer = C Rationale: Given the parent's descriptions of potential auditory hallucinations and possible paranoid thinking, it is important for the social worker to assess what is occurring with the parent's mental health by initiating a mental status exam (key C). A referral to a physician (A) or a meeting with the school (D) may be warranted but are not the NEXT steps. Option (B) assumes anxiety without a complete assessment.

A medical social worker discusses discharge planning with a patient's spouse. The patient's spouse is adamant that no follow-up medical care is needed. The patient has the capacity to make medical decisions. What should the social worker do NEXT? A. Respect the spouse's wishes B. Inform the patient's physician of the spouse's request C. Explain the patient's rights to the spouse D. Meet with the hospital board of ethics

Answer = C Rationale: Legal and ethical principles obligate the social worker to ensure that the spouse has a full understanding of the patient's rights (key C). The patient is competent, making (A) incorrect. Options (B) and (D) may be taken but would not be the NEXT action.

A social worker works in a community where gangs and drugs are a major problem. The mayor has requested that the social worker develop a plan involving the community to reduce gang violence and drug related problems. What should the social worker do FIRST? A. Create a forum for the gangs to meet B. Hire community leaders to lead the initiative C. Organize a task force to address the problems D. Conduct an educational workshop at the community center

Answer = C Rationale: Organizing a task force (key C) is the FIRST step in social action and involves those affected in addressing the issue and promoting solutions. Creating a forum (A) may result in promoting additional conflict and would not be indicated. Hiring community leaders (B) and conducting a workshop (D) are steps that could be taken later, but only after a task force is organized FIRST.

What components are included during the contracting phase of intervention? A. Informative events and critical incidents B. An ecomap and periods of observation C. Goals and objectives D. Assessment and evaluation

Answer = C Rationale: This item requires basic knowledge of the stages of intervention. Setting goals and objectives (key C) is a core component to the contracting phase of intervention.

A social worker is a member of the disaster response team in a community where there was a recent tornado. An elementary school was destroyed and three children were killed. A month later, parents and school officials ask the social worker to evaluate a child whose behavior has become unmanageable in class. The child is having difficulty concentrating and has not completed any assignments in several weeks. What should the social worker do FIRST? A. Refer the child for an evaluation for ADHD B. Encourage the parents to spend more time with the child each day C. Complete an individual assessment with the child D. Provide crisis intervention for the child

Answer = C Rationale: The child's behaviors are appropriate given the situation, so it is important to complete the individual assessment FIRST (key C) in order to determine the nature and extent of the issues. Options (A), (B), and (D) are all interventions that would follow the assessment.

What is the MOST important action that can be taken by a social work supervisor to reduce liability resulting from acts or omissions by supervisees? A. Advise the supervisees to communicate potential risks B. Request that supervisees sign waivers of liability C. Document topics discussed in supervision D. Purchase insurance that provides malpractice coverage

Answer = C Rationale: The documentation of topics covered in supervision (key C) is the MOST important action that a supervisor can take to reduce liability. Documentation will provide evidence of topics discussed and actions recommended. Option (A) relies on supervisees to take action, not the supervisor. While waivers of liability may be requested (B), this is not the MOST important action and does not waive liability. Option (D) is a reactive measure that does not reduce liability.

A social worker is providing therapy to a family. The husband and wife have been arguing constantly for several months and are considering a separation. Recently, the oldest child began acting out in school. With attention focused on the child's problems, arguments between the parents have subsided. What concept BEST describes this family's situation from a systems perspective? A. Reaction formation B. Adaptation C. Triangulation D. Entropy

Answer = C Rationale: The oldest child is exhibiting behaviors that is moving attention away from parental arguments. Triangulation (key C) is resulting in lessened parental conflict since the parents' attention has become focused on the child's behavior. Reaction formation (A) is a defense mechanism in which opposing tendencies are exaggerated. Adaptation (B) is a concept describing the ability of a system to adapt to change, which is not occurring. Entropy (D) is the opposite of adaptation and not the BEST description of the family dynamics described.

A social worker's research shows that when inmates who have been released from prison are given transportation resources, they are less likely to reoffend. The social worker submits the research to officials, advocating for a transportation program. What social work value BEST describes the social worker's efforts? A. Integrity in social work practice B. Inherent dignity and worth of individuals C. Promotion of social justice D. Competence in professional practice

Answer = C Rationale: The social worker in this item is attempting to rectify a social justice variable that is related to recidivism. Advocating for a broad transportation program is more strongly linked to concepts of social justice (key C) than to dignity and worth of individuals (B). Advocacy for the program is not related to upholding the integrity of social work practice (A) or professional competence (D).

A client is discharged from inpatient treatment for alcohol and prescription drug abuse. A mental health social worker meets for a follow-up session with family members and observes that they avoid discussing the problem. What should the social worker do FIRST? A. Confront family members about the long-term nature of the problem B. Assist the family in understanding the harm of alcohol and drug abuse C. Determine the extent of denial within the family system D. Advise the family members that relapse is a realistic possibility

Answer = C Rationale: The social worker should FIRST follow up on the observation that the family is using denial as a coping strategy (key C). This should occur before interventions such as confrontation (A), educating the family through helping their understanding of the harm involved (B), or explaining that relapse is possible (D).

A hospital social worker has a client who is noncompliant with management of diabetes. The client believes that the illness is God's will and God will provide a cure. What should the social worker do FIRST? A. Assure the client that God would want the client to be compliant B. Educate the client on the health consequences of being noncompliant C. Explore the role that religion plays in the client's overall self-care D. Inform the client that managing diabetes is not a spiritual issue

Answer = C Rationale: The social worker should strive to promote client self-determination and to respect differing values toward the role of religion. Understanding the role that religion plays (key C) is part of the assessment process. Options (A), (B), and (D) do not start where the client is and disregard his/her personal wishes. After FIRST exploring with the client the roles that religion plays in his or her belief system, educating the client on health consequences (B) may be an appropriate intervention.

Lithium would MOST likely be prescribed for what type of disorder? A. Sleep-wake B. Anxiety C. Bipolar I D. Dissociative

Answer = C Rationale: This is a basic recall question that requires discrimination of medication treatment for mental health disorders.

A social worker is attempting to secure public income support for an immigrant family. What does the social worker in this situation need to know? A. Immigrants are barred from receiving public assistance B. Only refugee immigrant population qualify for public assistance C. Eligibility varies for public assistance that is provided to immigrants D. Immigrants must become citizens to qualify for public assistance

Answer = C Rationale: This item requires a basic understanding of eligibility requirements for public assistance programs. It is important to know that eligibility varies depending on the program and jurisdiction (key C).

During an assessment, a social worker is trying to understand a client's perspective. At the same time, the social worker is maintaining the perspective of an outside observer. What concept is BEST reflected? A. Sympathy B. Supportive treatment C. Empathy D. Differential validation

Answer = C Rationale: This item requires an understanding of the concept of empathy (key C): attempting to understand the client from his or her perspective while maintaining objectivity.

A social worker receives a call from an angry client who yells at the social worker about perceived mistakes in a case. What should the social worker do NEXT? A. Suggest that the client discuss the concerns at the next session B. Refuse to listen because the client is behaving inappropriately C. Respond empathetically to the client's concerns D. Refer the client to the agency supervisor

Answer = C Rationale: To attend to the importance of the relationship between the social worker and the client, the social worker should demonstrate empathy NEXT (key C) before moving to action such as suggesting concerns be discussed at the next session (A), refusing to listen (B), and referring to the agency supervisor (D).

A social worker begins a job at a psychiatric hospital and is assigned a caseload. One of the clients is a former intimate partner. The social worker and client have not had any contact for several years. What should the social worker do? A. Process the issue with the client B. Deny the relationship occurred C. Transfer the client to a colleague D. Request a transfer to a different

Answer = C Rationale: the avoid the ethical dilemma of dual relationships, the social worker should transfer the client to another social worker (key C). Processing the issue with the client (A) is not going to resolve the ethical dilemma. Denying the existence of the past relationship (B) is unethical. Asking for a transfer to another department (D) avoids the issue entirely.

At what age does normal separation anxiety typically begin? A. 24 months B. 18 months C. 12 months D. 3 months

Answer = C Rationale: The item requires basic recall that separation anxiety normally occurs at the age of 12 months (key c).

A nine-month-old child is neglected and lacks a secure attachment to a caretaker. What long-term effect should the social worker be MOST concerned about as the child gets older? A. Posttraumatic Stress Disorder B. Social Anxiety in middle age C. Risk for incarceration D. Negative social interactions with peers

Answer = D Rational: The most strongly linked long-term effect of insecure attachment is future negative social interactions with peers (key D). Posttraumatic stress disorder (B) is a short-term effect, while risk of incarceration (C) and later social anxiety (D) are not directly linked to attachment difficulties.

A woman who is five months pregnant speaks with a social worker at an adoption agency to request information. The woman explains that she lives with her sister and that her sister's boyfriend is threatening to force her out of the home. She says that she does not feel safe. What should the social worker do FIRST? A. Call the police for the woman B. Develop a safety plan C. Try to locate housing for the woman D. Determine the nature of the danger

Answer = D Rationale: Before taking any action, the social worker should assess the situation FIRST (key D). Options (A), (B), and (C) are all action steps that may be done later. Taking action without assessing the situation involves reaching conclusions without knowing the facts.

A social worker employed by a retirement home is approached by a client for help in establishing a recycling program in the facility. The social worker decides to cohost a meeting with the client to discuss the idea with other residents. What is the MOST likely reason for this approach? A. Antagonizing administration could be dangerous. B. Legitimizing the residents' concerns may be helpful. C. Helping residents start new relationships is important. D. Sharing leadership is very effective with groups.

Answer = D Rationale: By sharing leadership with the resident (key D), the social worker is supporting the resident's skill development as well as promoting positive group dynamics. Options (A) and (B) draw conclusions without substantiated information. While the potential for beginning new relationships (C) exits. It is not the MOST likely reason for the co-facilitation.

A social worker is aware of a client having difficulty paying for prescription medications. What should the social worker do? A. Loan the client money to pay for the medications B. Suggest alternatives to prescription medications C. Inform the client of low-cost medications available online D. Encourage the client to talk with a pharmacist about payment options

Answer = D Rationale: Encouraging the client to talk with a pharmacist (key D) uses the expertise of that professional to review payment options such as generic drugs or pharmaceutical discount programs. Loaning the client money (A) does not address the long-term issue. Suggesting alternatives to medications (B) is outside the social worker's scope of practice. While providing information about low-cost online options (C) may be indicated, it is not likely that the social worker would be aware of all options in the same way that a pharmacist would.

A family services agency has a high number of clients who do not keep appointments. The agency's social work administrator conducts a client satisfaction survey, which reveals that clients believe the agency is not welcoming to their children. To alleviate the clients' concern, what should the administrator do FIRST? A. Create specialized activities for children B. Evaluate the need to change agency programs C. Obtain consultation to develop an action plan D. Form a committee of clients and staff to identify solutions

Answer = D Rationale: Engaging those who are directly involved is an important FIRST step. Forming a committee (key D) engages the clients and staff in formulating solutions. After forming the committee, options (A), (B), or (C) may follow, but they would not be done FIRST.

A social worker at a correctional facility is providing a treatment group for substance abusers who are inmates at the prison. A prison classification committee has selected 12 group participants of various racial and ethnic backgrounds. During the first three weeks of group, the social worker notices that very few participants seem willing to speak. What statement MOST likely explains the participants' reluctance to talk in the group? A. Inmates tend to exhibit much denial about substance abuse problems B. Racial tensions in prison populations inhibit communication in treatment situations C. Inmates often have an adversarial view of non-inmates, which can hinder disclosure D. Self-disclosure in group treatment contradicts behavioral expectations of prison culture

Answer = D Rationale: In a prison culture, the environmental influence does not promote participant self-disclosure (key D). Option (A) is an assumption that is stereotypical of prison inmates. Option (B) focuses on racial tension as one possible explanation, but it does not take into account the early stages of the group process and prison culture. There is not enough inforamtion provided to determine that option (C) is correct.

An adolescent who has been living with her 23-year-old boyfriend for six months calls a crisis hotline at 2:00 a.m. requesting assistance. She claims that her boyfriend evicted her because she may be pregnant. She will be 18 in two days. The adolescent says that she has not place to go. What should the social worker do FIRST? A. Obtain a court order for emergency foster care placement B. Notify the polic of the boyfriend's actions C. Call the adolescent's parents, requesting help D. Contact a youth shelter facility

Answer = D Rationale: In this crisis situation, the social worker must provide immediate, tangible aid. Contacting a youth shelter facility for housing FIRST (key D) helps with the clien'ts baskic need of shelter, which is the client's presenting problem. Notify the police (B) is not warranted by the boyfriend's actions since there is not indication a crime has been committed. Obtaining a court order for emergency placement (A) and calling the adolescent's parents (C) could possibly occur next if a youth shelter facility (key D) is not available.

An adolescent who has recently discharged from a psychiatric hospital is meeting with the school social worker to assist with school adjustment. The social worker would like to speak with the referring psychiatrist about the treatment plan. What should the social worker do FIRST? A. Advise the staff that specialized services will be required B. Review the student's records to determine if a discharge plan was sent C. Schedule a case conference to discuss the treatment plan D. Call the student's guardian to obtain a release of information

Answer = D Rationale: Informed consent from the student's guardian (key D) must be obtained FIRST before the social worker can speak with the psychiatrist. Advising the staff that specialized services will be required (A) is unrelated to the consultation with the psychiatrist. Reviewing records (B) does not respond to the social worker's desire to speak with the psychiatrist. A case conference (C) could be scheduled but only after a release of information is obtained (key D) FIRST.

A client is in treatment with a social worker with a fee-for-service setting. After paying in full and on time for the first ten sessions, the client begins to make either partial payments or no payments at all. Assessment of risk indicates that the client poses no danger to self or others. According to ethical standards of social work practice, what is the social worker's BEST response? A. Continue services after negotiating a nonmonetary compensation system B. Suspend services until the client has caught up on all financial obligations C. Terminate services after the third incident of insufficient or nonpayment D. Terminate services if debt persists after a discussion of financial obligations

Answer = D Rationale: Since the client poses no danger to self or others and is in arrears, services can be terminated after the discussion of financial obligations has occurred (key D). Negotiating a nonmentary compensation system (A) suggests bartering, which is not the BEST response. Suspending services until payment is made (B) is possible, but it is not the BEST response. Termination after a third time of nonpayment (C) uses an arbitrary number of incidents to make a decision after termination.

An adolescent is admitted to the hospital for the second time in three weeks for noncompliance with diabetes management. The physician believes that the adolescent is intentionally manipulating the diabetes care and wants to be admitted. The physician consults a social worker. What should the social worker do FIRST? A. Contract with the adolescent to prevent readmission B. Review the medical management plan with the adolescent C. Notify the local child welfare services D. Obtain a social and family history

Answer = D Rationale: The FIRST step for the social worker is to complete an assessment. Obtaining a social and family history (key D) is the only action that involves understanding the situation. Options (A), (B), and (C) may be necessary later, depending on what is learned during the assessment.

A social worker is contracted by a client whose initial complaint is depression. The client reports symptoms of insomnia, headaches, inability to eat, vomiting, and neck aches. What should the social worker do FIRST? A. Obtain a detailed social history on the client B. Probe for a substance abuse problem C. Engage the client is establishing goals D. Ask the client to obtain medical consultation

Answer = D Rationale: The described physical symptoms suggest a potential medical issue, which indicates the FIRST action is to obtain medical consultation (key D). Engaging the client in establishing goals (C) may be a step further along in the process, while obtaining a social history (A) and assessing for a substance abuse problem (B) may also be done later. It is important to FIRST rule out medical issues.

Parents of a 24-month-old child report their only concern is that their child plays alongside of, but not with, other children. What should the social worker do? A. Assess for symptoms of developmental delays B. Observe the child's play in a group setting C. Suggest that the parents report their concern at a doctor visit D. Reassure the parents that parallel play is typical at this age

Answer = D Rationale: The role of the social worker is to educate the child's parents about typical developmental behaviors (key D). This question requires a basic knowledge of parallel play.

A social worker meets with an adolescent child who wants to run away from home. The client gives several reasons for this desire. What is the BEST technique for the social worker to use? A. Confrontation B. Rationalization C. Exception exploration D. Partialization

Answer = D Rationale: The social worker needs to help the client gain a better understanding of the individual elements that are leading to the desire to fun away. By partializing (key D), issues are broken into manageable components that would facilitate further discussion and problem solving. Confrontation (A) and exception exploration (C) would be counterproductive. Rationalization (B) is a defense mechanism.

After several meetings with a client, a social worker notices the client making multiple complimentary statements about the social worker. What should the social worker do FIRST? A. Restate the purpose of the therapeutic relationship B. Ask the client to stop the compliments C. Confront the client about inappropriateness of the behavior D. Help the client explore the reasons for the behavior

Answer = D Rationale: The social worker should assess the reasons for the behavior FIRST (key D) before taking action. Restating the relationship's purpose (A), asking the client to stop (B), and confronting the client (C) may need to occur later, once the reasons for the behavior have been established.

During an initial interview with a social worker, a client reports a lack of desire for a sexual relationship with the spouse since getting married. The client engages in sex only when pressured, then dissociates during intercourse. The client has been having recurring violent nightmares and suicidal thoughts. What is the social worker's BEST response? A. Refer the client and spouse for sex therapy B. Refer the client to an inpatient psychiatric unit C. Request a neurological evaluation for the client D. Evaluate for a history of sexual abuse as a child

Answer = D Rationale: There is no reason to substantiate a referral to an inpatient psychiatric unit (B) since the client is displaying no symptoms necessitating that. There are no symptoms that suggest a neurological issue (C). Referring the client and spouse for sex therapy (A) would be premature, but evaluating past history (key D) is the BEST action for the social worker to take in order to determine the most appropriate treatment option for the client.

What term describes the belief that one's thoughts alone can result in the accomplishment of certain wishes? A. Depersonalization B. Magical thinking C. Thought broadcasting D. Looseness of associations

Answer = D Rationale: This is a basic recall item requiring test-taker knowledge of commonly used terms related to cognition.

What characteristic is one of the diagnostic criteria of hoarding disorder? A. Inappropriate compensatory behavior B. Unmanaged stress C. Avoidant personality traits D. Impaired quality of life

Answer = D Rationale: This is a recall question that requires knowledge that the diagnostic criteria for hoarding disorder include impaired quality of life (key D).

A social worker assesses a victim of psychological abuse by a long-term partner. What is MOST important for the social worker to remember about the dynamics of abuse within a relationship? A. Abuse frequently arises from uncontrolled anger B. Abuse is often a response to the fear of abandonment C. Abuse frequently comes from feelings of inadequacy caused by jealousy D. Abuse is often an effort to retain control

Answer = D Rationale: This item is a basic recall question of the dynamics of abuse. While (A), (B), and (C) may occur, retaining control (key D) is the MOST common underlying dynamic in abusive relationships.

A social worker is working with a client who is severely depressed. The social worker has learned that earlier in the year the client boasted to others of his perceived importance, went without sleep for several days, spent large amounts of money, and talked rapidly. What disorder is the MOST likely diagnosis for this client? A. Schizoaffective B. Cyclothymic C. Persistent depressive D. Bipolar I

Answer = D Rationale: This item is a basic recall question of the indicators of bipolar I disorder. Persistent depressive disorder (C) features low mood, while cyclothymic disorder (B) features numerous periods of changes in mood. The vignette describes one episode, not several. Option (A) is not indicated, based on the information provided. Bipolar I Disorder (key D) is the MOST likely diagnosis because one occurrence of the mood change is noted rather than multiple occurrences.

What is the MOST likely reason a client in an outpatient alcohol treatment program would be prescribed the drug naltrexone in conjunction with counseling? A. To treat clinical depression B. To ease alcohol withdrawal symptoms C. To induce necessary sleep D. To decrease cravings for alcohol

Answer = D Rationale: This item requires a basic knowledge of alcohol treatment and interventions.

The relationship with a client reminds a social worker of the social worker's relationship with a sibling. This is an example of: A. Transference B. A therapeutic block C. A dual relationship D. Countertransference

Answer = D Rationale: This item requires a basic understanding of the definition of countertransference (key D).

A social worker at a community mental health program includes a spiritual assessment as part of the intake interview. The program serves members of a local tribal community. What is important for the social worker to remember when working with clients who are Native Americans/Indigenous people? A. Closed-ended questions within a structured protocol should be used B. The assessment must be kept brief and storytelling minimized C. Clinical language should be used to create a safe environment D. Trust should be established before asking for sensitive information

Answer = D Rationale: This item requires an understanding of the cultural aspects of working with Native American/Indigenous peoples communities. Trust must be established before asking for sensitive information (key D). The use of clinical language (C) and closed-ended questions (A) are barriers to building trust. Because storytelling is a cultural tradition in these communities, storytelling should be included in interventions and no minimized, making option (B) incorrect.

A social worker disagrees with the way agency policy restricts client services and believes it is an ethical issue. What should the social worker do NEXT? A. Inform clients of the ethical disagreement B. Identify possible actions to resolve the issue C. Organize colleagues to lobby for change D. Determine whether ethical values are in conflict

Answer = D Rationale: This item requires basic understanding of the process involving steps for ethical problem solving. The NEXT step is to determine whether ethical values are in conflict (key D).

A social worker assesses a client with an eating disorder. The client expresses ambivalence about changing eating habits. What should the social worker do NEXT? A. Advocate for change B. Confront the resistance C. Explore the client's values D. Validate the client's concerns

Answer = D Rationale: This item requires knowledge of the features of client ambivalence. When advocating for change (A), confronting the resistance (B), and exploring the client's values (C) may be appropriate actions, the NEXT step is to validate the client's concerns (key D).

A social worker provides opportunities to community residents to increase political participation. The goal is to affect policy and improve socioeconomic conditions. What approach is the social worker MOST likely using? A. Networking B. Program development C. Coalition building D. Empowerment

Answer = D Rationale: This item requires knowledge of the function of empowerment (key D), which is most strongly associated with policy and socioeconomic change. Networking (A) and coalition building (C) are specific actions and not approaches. Program development (B) is more closely associated with organizational growth and change, not community-level actions.

To take into account the information processing deficits of clients with schizophrenia, what should the social worker do to BEST structure group treatment? A. Encourage the free flow of ideas B. Use expressive therapy tools C. Enforce a rigid agencda for the group D. Schedule a variety of brief activities

Answer = D Rationale: This item requires understanding of schizophrenia and the information processing deficits that are typically present. Because of the characteristics of schizophrenia, encouraging the free flow of ideas (A), use of expressive therapy tools (B), and enforcing a rigid group agenda (C) are not indicated. Providing a schedule of brief activities (key D) is BEST suited for clients with schizophrenia.

What condition is NOT a cuase of mild or major neurocognitive disorder? A. Alzheimer's disease B. Substance abuse C. Cerebral vascular event D. Osteoporosis

Answer = D Rationale: This itme requires knowledge that osteoporosis (key D) is not directly linked to mile or major neurocognitive disorders. Options (A), (B), and (C) are each correlated with neurocognitive disorders.

In a small community hospital, the newly hired social worker learns that some nurses document that they perform a psychosocial family assessment as part of the patient's admission history. The social worker proposes making this a social work function. What is the BEST approach to advocate for this proposal? A. Document how much time the change will save the staff B. Compare length of stay data for each approach C. Ask the nurses why they are doing the family assessments D. Identify the unique skills and abilities of social workers

Answer = D Rationale: To BEST advocate for this proposal with the nursing staff, the social worker will need to educate them on social work skills and abilities (key D). Comparing length of stay data (B) and asking why nurses are completing family assessments (C) both have the potential of putting the nursing staff on the defensive and are not the BEST approach to successfully advocate for this proposal. Documenting how much time the change will save staff (A) is not an approach to advocacy.

A new social worker receives a court order to disclose client records. The social worker notes that there are grammatical problems and minor errors regarding dates and event in the recordings. What should the social worker do FIRST? A. Make corrections as needed B. Claim record confidentiality C. Send the records D. Meet with a supervisor

Answer = D Rationale: When a court order is received to disclose client records, supervision should be obtained FIRST (key D). Depending on the circumstances, the records may need to be sent (C) or a claim for confidentiality may need to be made (B). Making altercations or corrections to the record (A) should not be done.

A social worker is in the early stages of intervention with a client who is acutely depressed. What approach style would be MOST effective for the social worker to use at this point in the treatment process? A. Emotive B. Directive C. Motivational D. Supportive

Answer = D Rationale: Working with a newer client who is acutely depressed requires a supportive approach (key D). The nature of the depression contraindicates emotive (A), directive (B), or motivational (C) approaches.

A social worker meets with a client who reports shortness of breath and a rapid heartbeat. The client has a fear of losing control. What is the MOST likely explanation for the client's condition? A. Impulse control disorder B. Delusional disorder C. Neurocognitive disorder D. Panic disorder

Answer = D Rationale:The symptoms described by the client are MOST indicative of panic disorder (key D).

A 16-year-old client who is depressed mentions suicidal plans to a social worker. Privacy and confidentiality have been discussed early in the treatment process. The social worker believes that the threat is serious. What should the social worker do FIRST? A. Respect the client's right to confidentiality B. Consult with an agency supervisor about the dilemma C. Discuss a safety contract with the client D. Arrange for the client to receive a crisis assessment

Answer = D Rationale: Arranging for a crisis assessment (key D) is the NEXT step after the social worker has determined the existence of a serious threat. Consulting with a supervisor (B) would be appropriate but only after arranging for a crisis assessment (key D). Rights to confidentiality (A) are superseded by the seriousness of the threat. A safety contract (C) may be appropriate after an assessment is completed.

A social worker completes an initial assessment of a client suffering from anorexia nervosa with significant weight loss. To BEST meet the client's immediate needs, what should the social worker do? A. Network with a nutritionist who can recommend nutritional supplementation B. Refer the client to an occupational therapist who can model beneficial exercises C. Consult with a physician regarding possible hospitalization D. Contact a registered dietician who can teach the client healthy eating habits

Given the severity of the weight loss, it is imperative that a consultation with a physician occur (key C) to address the client's immediate needs. While contact with other disciplines (A), (B), and (D) may be helpful, a medical consultation is the BEST response.


Related study sets

Azure Infrastructure and Networking

View Set

Microeconomics -- Chapter 8 (Part 1): Short-Run Costs and Output Decisions

View Set